The blue dot is at what value on the number line?

The Blue Dot Is At What Value On The Number Line?

Answers

Answer 1

Answer:

-19

Step-by-step explanation:

By looking at the 2 numbers provided, -10 and -4, you can work out that there is a gap of 6 numbers as(-4) - (-10) = 6

There are 2 intervals between -10 and -4, so each interval is

6/2 = 3

a gap of 3

This means the number to the left of -4 is -7, then -10 which works.

From there, you count how many intervals there is between -10 and the ?

There are 3 intervals, so you have to decrease -10 by -3x3 or -9

Therefore the ? is -19

Another way is to just count it directly

The number directly left of -10 is going to be -13, then -16 and finally -19


Related Questions

Two sides of a triangle are equal length. The length of the third side exceeds the length of one of the other sides by 3 centimeters. The perimeter of the triangle is 93 centimeters. Find the length of each of the shorter sides of the triangle

Answers

Answer:

30 cm

Step-by-step explanation:

let x be the lenght of the two sides of equal lenghts, so the other is x+3

and the perimeter is x+x +x +3

P=3x+3

P=3(x+1)

93=3(x+1)

31=x+1

x=30

so the shorter sides are of 30 centimeters and the longest is 33

A simple random sample of 28 Lego sets is obtained and the number of pieces in each set was counted.The sample has a standard deviation of 12.65. Use a 0.05 significance level to test the claim that the number of pieces in a set has a standard deviation different from 11.53.

Answers

Answer:

Step-by-step explanation:

Given that:

A simple random sample n = 28

sample standard deviation S = 12.65

standard deviation [tex]\sigma[/tex] = 11.53

Level of significance ∝ = 0.05

The objective is to test the claim that the number of pieces in a set has a standard deviation different from 11.53.

The null hypothesis and the alternative hypothesis can be computed as follows:

Null hypothesis:

[tex]H_0: \sigma^2 = \sigma_0^2[/tex]

Alternative hypothesis:

[tex]H_1: \sigma^2 \neq \sigma_0^2[/tex]

The test statistics can be determined by using the following formula in order to test if the claim is statistically significant or not.

[tex]X_0^2 = \dfrac{(n-1)S^2}{\sigma_0^2}[/tex]

[tex]X_0^2 = \dfrac{(28-1)(12.65)^2}{(11.53)^2}[/tex]

[tex]X_0^2 = \dfrac{(27)(160.0225)}{132.9409}[/tex]

[tex]X_0^2 = \dfrac{4320.6075}{132.9409}[/tex]

[tex]X_0^2 = 32.5002125[/tex]

[tex]X^2_{1- \alpha/2 , df} = X^2_{1- 0.05/2 , n-1}[/tex]

[tex]X^2_{1- \alpha/2 , df} = X^2_{1- 0.025 , 28-1}[/tex]

From the chi-square probabilities table at 0.975 and degree of freedom 27;

[tex]X^2_{0.975 , 27}[/tex] = 14.573

[tex]X^2_{\alpha/2 , df} = X^2_{ 0.05/2 , n-1}[/tex]

[tex]X^2_{\alpha/2 , df} = X^2_{0.025 , 28-1}[/tex]

From the chi-square probabilities table at 0.975 and degree of freedom 27;

[tex]X^2_{0.025 , 27}=[/tex] 43.195

Decision Rule: To reject the null hypothesis if [tex]X^2_0 \ > \ X^2_{\alpha/2 , df} \ \ \ or \ \ \ X^2_0 \ < \ X^2_{1- \alpha/2 , df}[/tex] ; otherwise , do not reject the null hypothesis:

The rejection region is [tex]X^2_0 \ > 43.195 \ \ \ or \ \ \ X^2_0 \ < \ 14.573[/tex]

Conclusion:

We fail to reject the null hypothesis since  test statistic value 32.5002125  lies  between 14.573 and 43.195.

It takes amy 8 minutes to mow 1/6 of her backyard. At that rate how many more minutes will it take her to finish mowing her backyard

Answers

Answer:

40 minutes

Step-by-step explanation:

If it takes her 8 minutes to mow 1/6 of it, we can find the total amount of time it  will take by multiplying 8 by 6, since 1/6 times 6 is 1 (1 represents the whole lawn mowed)

8(6) = 48

The question asks for how many more minutes it will take, so subtract 48 by 8.

48 - 8 = 40

= 40 minutes

Answer:

40 minutes

Step-by-step explanation:

We can use ratios to solve

8 minutes          x minutes

------------------- = ----------------

1/6 yard                 1 yard

Using cross products

8 * 1 = 1/6 x

Multiply each side by 6

8*6 = 1/6 * x * 6

48 = x

48 minutes total

She has already done 8 minutes

48-8 = 40 minutes

According the the U.S. Department of Education, full-time graduate students receive an average salary of $15,000 with a standard deviation of $1,200. The dean of graduate studies at a large state university in PA claims that his graduate students earn more than this. He surveys 100 randomly selected students and finds their average salary is $16,000. Use a significance level of 0.05 to test if there is evidence that the dean's claim is correct. What are the hypotheses

Answers

Answer:

Step-by-step explanation:

Given that :

population Mean = 15000

standard deviation= 1200

sample size n = 100

sample mean = 16000

The null and the alternative hypothesis can be computed as follows:

[tex]\mathtt{H_o : \mu = 15000 }\\ \\ \mathtt{H_1 : \mu > 15000}[/tex]

Using the standard normal z statistics

[tex]z = \dfrac{\overline X - \mu}{\dfrac{\sigma }{\sqrt{n}}}[/tex]

[tex]z = \dfrac{16000 -15000}{\dfrac{1200 }{\sqrt{100}}}[/tex]

[tex]z = \dfrac{1000}{\dfrac{1200 }{10}}[/tex]

[tex]z = \dfrac{1000\times 10}{1200}[/tex]

z = 8.333

degree of freedom = n - 1 = 100 - 1 = 99

level of significance ∝ = 0.05

P - value from the z score = 0.00003

Decision Rule: since the p value is lesser than the level of significance, we reject the null hypothesis

Conclusion: There is sufficient evidence  that the Dean claim for his graduate students earn more than average salary of $15,000

Dean's Claim of Average Salary = 16000, ie greater than 15000 : is correct

Null Hypothesis [ H0 ] : Average Salary = 15000

Alternate Hypothesis [ H1 ] : Average Salary > 15000

Hypothesis is tested using t statistic.

t = ( x - u ) / ( s / √ n ) ; where -

x = sample mean , u = population mean , s = standard deviation, n = sample size

t = ( 16000 - 15000 ) / ( 1200 / √100 )

= 1000 / 120

t  {Calculated} = 8.33,

Degrees of Freedom = n - 1 = 100 = 1 = 99

Tabulated t 0.05 (one tail) , at degrees of freedom 99 = 1.664

As Calculated t value 8.33 > Tabulated t value 1.664 , So we reject the Null Hypothesis in favour of Alternate Hypothesis.

So, conclusion : Average Salary > 15000

To learn more, https://brainly.com/question/17099835?referrer=searchResults

The value of y varies jointly with x and z. If y = 2 when z = 110 and x = 11, find the approximate value of y when x = 13 and z = 195.

Answers

Answer:

y = 4

Step-by-step explanation:

To find the approximate value of y when

x = 13 and z = 195 we must first find the relationship between them

The statement

y varies jointly with x and z is written as

y = kxz

where k is the constant of proportionality

From the question

y = 2

x = 11

z = 110

We have

2 = 11(110)k

2 = 1210k

Divide both sides by 1210

[tex]k = \frac{1}{605} [/tex]

So the formula for the variation is

[tex]y = \frac{1}{605} xz[/tex]

When

x = 13

z = 195

y is

[tex]y = \frac{1}{605} (13)(195)[/tex]

[tex]y = \frac{507}{121} [/tex]

y = 4.1900

We have the final answer as

y = 4

Hope this helps you

Graph the following set of parametric equations on your calculator and select the matching graph.

Answers

Answer:

Graph 2

Step-by-step explanation:

As you can see the first equation is present with a negative slope, and none of the graphs have a line plotted with a negative slope, besides the second graph. That is your solution.

Explain how to solve the inequality (x + 1)(x – 2) ∙ (x – 3) > 0. Explain in your own words, each step necessary to solve the inequality, making sure to follow the proper order of operations. Is this inequality accurate? Explain why or why not.

Answers

Answer:

[tex]x > -1[/tex] or

[tex]x > 2[/tex] or

[tex]x > 3[/tex]

Step-by-step explanation:

Given

[tex](x + 1)(x - 2) (x - 3) > 0[/tex]

Required

Solve; with steps

[tex](x + 1)(x - 2) (x - 3) > 0[/tex]

Start by splitting the inequality as follows

[tex]x + 1 > 0[/tex] or [tex]x - 2 > 0[/tex] or  [tex]x - 3 > 0[/tex]

Solve the inequalities one after the other

Solving: [tex]x + 1 > 0[/tex]

Subtract 1 from both sides

[tex]x + 1 - 1 > 0 - 1[/tex]

[tex]x > -1[/tex]

Solving: [tex]x - 2 > 0[/tex]

Add 2 to both sides

[tex]x - 2 +2 > 0 +2[/tex]

[tex]x > 2[/tex]

Solving: [tex]x - 3 > 0[/tex]

Add 3 to both sides

[tex]x - 3 +3> 0+3[/tex]

[tex]x > 3[/tex]

Hence, the solution to the inequality is

[tex]x > -1[/tex] or

[tex]x > 2[/tex] or

[tex]x > 3[/tex]

Compute (3/4)*(8/9)*(15/16)*(24/25)*(35/36)*(48/49)*(63/64)*(80/81)*(99/100) Express your answer in the simplest way possible. (Suggestion: First, try computing 3/4*8/9 then 3/4*8/9*15/16 and so on. Look for patterns.

Answers

Answer:

[tex](\frac{3}{4})*(\frac{8}{9})*(\frac{15}{16})*(\frac{24}{25})*(\frac{35}{36})*(\frac{48}{49})*(\frac{63}{64})*(\frac{80}{81})*(\frac{99}{100}) = \frac{11}{20}[/tex]

Step-by-step explanation:

Given

[tex](\frac{3}{4})*(\frac{8}{9})*(\frac{15}{16})*(\frac{24}{25})*(\frac{35}{36})*(\frac{48}{49})*(\frac{63}{64})*(\frac{80}{81})*(\frac{99}{100})[/tex]

Required

Simplify

For clarity, group the expression in threes

[tex]((\frac{3}{4})*(\frac{8}{9})*(\frac{15}{16}))*((\frac{24}{25})*(\frac{35}{36})*(\frac{48}{49}))*((\frac{63}{64})*(\frac{80}{81})*(\frac{99}{100}))[/tex]

Evaluate the first group [Divide 8 by 4]

[tex]((\frac{3}{1})*(\frac{2}{9})*(\frac{15}{16}))*((\frac{24}{25})*(\frac{35}{36})*(\frac{48}{49}))*((\frac{63}{64})*(\frac{80}{81})*(\frac{99}{100}))[/tex]

[Divide 9 by 3]

[tex]((\frac{1}{1})*(\frac{2}{3})*(\frac{15}{16}))*((\frac{24}{25})*(\frac{35}{36})*(\frac{48}{49}))*((\frac{63}{64})*(\frac{80}{81})*(\frac{99}{100}))[/tex]

[tex]((\frac{2}{3})*(\frac{15}{16}))*((\frac{24}{25})*(\frac{35}{36})*(\frac{48}{49}))*((\frac{63}{64})*(\frac{80}{81})*(\frac{99}{100}))[/tex]

[Divide 15 by 3]

[tex]((\frac{2}{1})*(\frac{5}{16}))*((\frac{24}{25})*(\frac{35}{36})*(\frac{48}{49}))*((\frac{63}{64})*(\frac{80}{81})*(\frac{99}{100}))[/tex]

[Divide 16 by 2]

[tex]((\frac{1}{1})*(\frac{5}{8}))*((\frac{24}{25})*(\frac{35}{36})*(\frac{48}{49}))*((\frac{63}{64})*(\frac{80}{81})*(\frac{99}{100}))[/tex]

[tex](\frac{5}{8})*((\frac{24}{25})*(\frac{35}{36})*(\frac{48}{49}))*((\frac{63}{64})*(\frac{80}{81})*(\frac{99}{100}))[/tex]

Evaluate the second group [Divide 35 and 25 by 5]

[tex](\frac{5}{8})*((\frac{24}{5})*(\frac{7}{36})*(\frac{48}{49}))*((\frac{63}{64})*(\frac{80}{81})*(\frac{99}{100}))[/tex]

[Divide 49 by 7]

[tex](\frac{5}{8})*((\frac{24}{5})*(\frac{1}{3})*(\frac{4}{7}))*((\frac{63}{64})*(\frac{80}{81})*(\frac{99}{100}))[/tex]

[Divide 24 by 3]

[tex](\frac{5}{8})*((\frac{8}{5})*(\frac{1}{1})*(\frac{4}{7}))*((\frac{63}{64})*(\frac{80}{81})*(\frac{99}{100}))[/tex]

[tex](\frac{5}{8})*((\frac{8}{5})*(\frac{4}{7}))*((\frac{63}{64})*(\frac{80}{81})*(\frac{99}{100}))[/tex]

Merge the first and second group

[tex]((\frac{5}{8})*(\frac{8}{5})*(\frac{4}{7}))*((\frac{63}{64})*(\frac{80}{81})*(\frac{99}{100}))[/tex]

[tex](1*(\frac{4}{7}))*((\frac{63}{64})*(\frac{80}{81})*(\frac{99}{100}))[/tex]

[tex](\frac{4}{7})*((\frac{63}{64})*(\frac{80}{81})*(\frac{99}{100}))[/tex]

Evaluate the last group [Divide 99 by 9]

[tex](\frac{4}{7})*((\frac{63}{64})*(\frac{80}{9})*(\frac{11}{100}))[/tex]

[Divide 63 by 9]

[tex](\frac{4}{7})*((\frac{7}{64})*(\frac{80}{1})*(\frac{11}{100}))[/tex]

[Divide 64 and 80 by 8]

[tex](\frac{4}{7})*((\frac{7}{8})*(\frac{10}{1})*(\frac{11}{100}))[/tex]

[Divide 10 and 4 by 2]

[tex](\frac{4}{7})*((\frac{7}{4})*(\frac{5}{1})*(\frac{11}{100}))[/tex]

[Divide 100 by 5]

[tex](\frac{4}{7})*((\frac{7}{4})*(\frac{1}{1})*(\frac{11}{20}))[/tex]

[tex](\frac{4}{7})*((\frac{7}{4})*(\frac{11}{20}))[/tex]

[tex](\frac{4}{7})*(\frac{7}{4})*(\frac{11}{20})[/tex]

[tex]1*(\frac{11}{20})[/tex]

[tex]\frac{11}{20}[/tex]

Hence;

[tex](\frac{3}{4})*(\frac{8}{9})*(\frac{15}{16})*(\frac{24}{25})*(\frac{35}{36})*(\frac{48}{49})*(\frac{63}{64})*(\frac{80}{81})*(\frac{99}{100}) = \frac{11}{20}[/tex]

Find the interest on a Principal Balance of $10,000 over the course of eight years with an interest rate of 5.5%. Do this for: Simple Interest.

Answers

Answer:

Simple Interest : $ 4400

Step-by-step explanation:

We want to calculate the interest on $ 10,000, at 5.5% interest rate per year, over a course of 8 years.

We can use the simple interest formula here, or :

I = P × r × t,

Where P is the principle amount, $ 10,000, r is the interest rate, 5.5% each year, or in decimal form 5.5 / 100 = 0.055. t is the time, 8 years.

Simple Interest : 10000 × 0.055 × 8 =  $4400.00

Then again the interest can be added to the principal amount ( $10,000 ) to receive some new amount after 8 years, which is $ 14,000. However the simple interest earned in 8 years at a rate of 5.5% should be $4400.

The simple interest earned on the amount is $4,400

Interest is the total amount that would be paid or earned from making an investment or taking a loan over a period of time.

Simple Interest  = principal x time x interest rate

principal = amount borrowed = $10,000

time = 8 years

Interest rate = 5.5%

10,000 x 0.055 x 8 = $4,400

To learn more about simple interest, please check: https://brainly.com/question/9352088?referrer=searchResults

two ratios equivalent to 27:9

Answers

Answer:

Those ratios could be 3:1

find the area of square whose side is 2.5 cm

Answers

Answer:

6.25

Step-by-step explanation:

2.5 *2.5=6.25

Answer:

6.25cm^2.

Step-by-step explanation:

To find the area of a square, you multiply the two sides, 2.5✖️2.5.

This gives the area of 6.25cm^2.

Hope this helped!

Have a nice day:)

|5x|=3 please help me

Answers

Answer: see below

Explanation:

|5x| = 3

5x = 3
x = 3/5

5x = -3
x = -3/5

Bianca took a job that paid $150 the first week. She was guaranteed a raise of 6% each week. How much money will she make in all over 8 weeks? Round the answer to the nearest cent. please answer with the reasoning, I want to learn how to solve this and not just get the answer. Thank you.

Answers

Answer:

$225.54 (hope it help)

Step-by-step explanation:

for 2nd week

$150 for the first week and a raise of 6% each week

which means 150+6%

6% of 150 is 9 (150x0.06)

150+9=159

and it repeats

for 3rd week

6% of 159 is 9.54 (159x0.06)

159+9.54=168.54

for 4th week

6% of 168.54 is 10.1124 (168.54x0.06)

168.54+10.1124=178.652

for 5th week

6% of 178.652 is 10.71912 (178.652x0.06)

178.652+10.71912=189.37112

an easier to do it is to just do 178.652 + 6% on your calculater

and I'll skip all the way to the 8th since you know the formula

212.777390432+6%=225.544033858

225.544033858≈225.54

The quotient of 8 and the difference of three and a number​.
Answer: 8÷(3-x)

Answers

Answer:

Below

Step-by-step explanation:

● 8 ÷ (3-x)

Dividing by 3-x is like multiplying by 1/(3-x)

● 8 × (1/3-x)

● 8 /(3-x)

A research center claims that ​% of adults in a certain country would travel into space on a commercial flight if they could afford it. In a random sample of adults in that​ country, ​% say that they would travel into space on a commercial flight if they could afford it. At ​, is there enough evidence to reject the research

Answers

Complete Question

A research center claims that ​30% of adults in a certain country would travel into space on a commercial flight if they could afford it. In a random sample of 700 adults in that​ country, ​34% say that they would travel into space on a commercial flight if they could afford it. At ​, is there enough evidence to reject the research center's claim

Answer:

Yes there is  sufficient evidence to reject the research center's claim.

Step-by-step explanation:

From the question we are told that

     The population proportion is  p = 0.30

      The sample proportion is  [tex]\r p = 0.34[/tex]

       The  sample size is  n = 700

The null hypothesis is  [tex]H_o : p = 0.30[/tex]

 The  alternative hypothesis is  [tex]H_a : p \ne 0.30[/tex]

Here we are going to be making use of  level of significance  =  0.05 to carry out this test

Now we will obtain the critical value of  [tex]Z_{\alpha }[/tex] from the normal distribution table , the value is  [tex]Z_{\alpha } = 1.645[/tex]

 Generally the test statistics is mathematically represented as

            [tex]t = \frac{ \r p - p }{ \sqrt{ \frac{ p (1-p)}{n} } }[/tex]

substituting values

              [tex]t = \frac{ 0.34 - 0.30 }{ \sqrt{ \frac{ 0.30 (1-0.30 )}{ 700} } }[/tex]

              [tex]t = 2.31[/tex]

Looking at the values of t  and  [tex]Z_{\alpha }[/tex] we see that [tex]t > Z_{\alpha }[/tex] hence the null hypothesis is rejected

 Thus we can conclude that there is  sufficient evidence to reject the research center's claim.

88 feet/second = 60 miles/hour. How many feet per second is 1 mile/hour? (Hint: divide both sides of the equation
by the same amount.)
Round to the nearest thousandth.
One mile per hour is equivalent to
ao feet/second

Answers

Answer: 1ft/sec = 0.618 mi/hr

Explanation:

88 ft/sec = 60 mi/hr
88/88 ft/sec = 60/88 mi/hr (divide both sides by 88)
1 ft/sec = 60/88 mi/hr
1 ft/sec = 15/22 mi/hr
1 ft/sec = 0.681 mi/hr

Find (fºg)(2) and (f+g)(2) when f(x)= 1/x and g(x) = 4x +9

Answers

[tex](f\circ g)(2)=\dfrac{1}{4\cdot2+9}=\dfrac{1}{17}\\\\(f+g)(2)=\dfrac{1}{2}+4\cdot2+9=\dfrac{1}{2}+17=\dfrac{1}{2}+\dfrac{34}{2}=\dfrac{35}{2}[/tex]

Max believes that the sales of coffee at his coffee shop depend upon the weather. He has taken a sample of 5 days. Below you are given the results of the sample.
Cups of Coffee Sold Temperature
350 50
200 60
210 70
100 80
60 90
40 100
A. Which variable is the dependent variable?
B. Compute the least squares estimated line.
C. Compute the correlation coefficient between temperature and the sales of coffee.
D. Predict sales of a 90 degree day.

Answers

Answer:

1. cups of coffee sold

2.Y = 605.7 - 5.943x

3. -0.952

4. 70.84

Step-by-step explanation:

1. the dependent variable in this question is the cups of coffee sold

2. least square estimation line

Y = a+bx

we have y as the cups of coffee sold

x as temperature.

first we will have to solve for a and then b

∑X = 450

∑Y = 960

∑XY = 61600

∑X² = 35500

∑Y² = 221800

a = ∑y∑x²-∑x∑xy/n∑x²-(∑x)²

a = 960 * 35500-450*61600/6*35500-450²

a = 6360000/10500

= 605.7

b = n∑xy - ∑x∑y/n∑x²-(∑x)²

= 6*61600 - 450*960/6*35500 - 450²

= -5.943

the regression line

Y = a + bx

Y = 605.7 - 5.943x

3. we are to find correlation coefficient

r = n∑xy - ∑x∑y multiplied by√(n∑x²-(∑x)² * (n∑y² - (∑y)²)

= 6*61600 -960*450/√(6*35500 - 450²)*(6*221800 - 960²)

=-62400/√4296600000

= -62400/65548.5

= -0.952

4. we have to predict sales of a 90 degree day fro the regression line

Y = 605.7 - 5.943x

y = 605.7 - 5.943(90)

y = 605.7 - 534.87

= 70.84

Peter saved up $20,000 in an account earning a nominal 5% per year compounded continuously. How much was in the account at the end of two years? Round the answer to nearest dollar.

Answers

Answer: 22,103

Step-by-step explanation:

Compound interest is the interest calculated on the initial principal and the accumulated interest.

The amount in the account at the end of two years is $22,050.

What is compound interest?

Compound interest is the interest calculated on the initial principal and the accumulated interest.

We have,

Principal = $20,000

Rate = r = 5%

It is compounded yearly.

Time = t = 2 years.

The formula for the amount having compound interest:

A = P [tex]( 1 + \frac{r}{n} )^{nt}[/tex]

A = 20,000 [tex](1 + \frac{5}{100\times1})^{2\times1}[/tex]

A = 20,000 ( 1 + 5/100 )²

A = 20,000 ( 105/100 )²

A = (20,000 x 105 x 105) / (100 x 100)

A = 2 x 105 x 105

A = $22,050

Thus the amount in the account at the end of two years is $22,050.

Learn more about compound interest here:

https://brainly.com/question/14740098

#SPJ2

What is the domain of the set of ordered pairs?
(8, -13); ( 0,-5); (4, -9); (-3,2)

Answers

The domain is the input values, which are the x values.

The x values in the given pairs are: 8, 0,4,-3

The domain set is (-3, 0, 4, 8)

The required domain of the set of ordered pairs is [8, 0, 4, -3]

Given that,

Set of ordered pair; (8, -13); ( 0,-5); (4, -9); (-3,2).

We have to determine,

The domain of the set of ordered pair.

According to the question,

The domain refers to the set of possible input values.

The domain of a graph consists of all the input values shown on the x-axis.

A relation is a set of ordered pairs.

The domain is the set of all the first components of the ordered pairs.

Then,

Set of ordered pair; (8, -13); ( 0,-5); (4, -9); (-3,2).

Here, Set of all the input values on the x-axis.

Therefore,

The set of values of x is { 8,0,4,-3 }

Hence, The required domain of the set of ordered pairs is [8, 0, 4, -3]

To know more about Domain click the link given below.

https://brainly.com/question/19704059

Consider population data with μ = 30 and σ = 3. (a) Compute the coefficient of variation. (b) Compute an 88.9% Chebyshev interval around the population mean. Lower Limit Upper Limit

Answers

Answer:

A. 10%

B. Lower limit= 21

Upper limit = 39

Step-by-step explanation:

Mean = 30

SD = 3

a. COV = SD/|x| × 100

= 3/30 × 100

= 10%

= 0.1

B. For 88.9 chevbychev interval:

= (1 - 1/K²) = 0.889

= 1/K² = 1 - 0.889

= 1/K² = 0.111

= K² = 1/0.111

= K² = 9

= K = √9

K = 3

Lower limit = 30 - 3(3)

Lower limit = 21

Upper limit = 30 + 3(3)

Upper limit = 39

Therefore lower limit is 21 and upper limit is 39

If the normality requirement is not satisfied​ (that is, ​np(1​p) is not at least​ 10), then a​ 95% confidence interval about the population proportion will include the population proportion in​ ________ 95% of the intervals. ​(This is a reading assessment question. Be certain of your answer because you only get one attempt on this​ question.)

Answers

Answer:

less than

Step-by-step explanation:

If the normality requirement is not satisfied​ (that is, ​np(1​ - p) is not at least​ 10), then a​ 95% confidence interval about the population proportion will include the population proportion in​ _less than__ 95% of the intervals.

The confidence interval consist of all reasonable values of a population mean. These are value for which the null hypothesis will not be rejected.

So, let assume that If the 95%  confidence interval contains the value for the hypothesized mean, then the sample mean  is reasonably close to the hypothesized mean. The effect of this is that the p- value is going to be greater than 0.05, so we fail to reject the null hypothesis.

On the other hand,

If the 95%  confidence interval do not contains the value for the hypothesized mean, then the sample mean  is far away from the hypothesized mean. The effect of this is that the p- value is going to be lesser than 0.05, so we reject the null hypothesis.

If tanA = 3
evaluate
CosA + sinA\
casA - SinA​

Answers

Answer:

Hi, there!!!

I hope you mean to evaluate cosA+ sonA /cosA - sinA.

so, i hope the answer in pictures will help you.

The dot plot represents a sampling of ACT scores: dot plot titled ACT Scores with Score on the x axis and Number of Students on the y axis with 1 dot over 24, 3 dots over 26, 3 dots over 27, 5 dots over 28, 3 dots over 30, 3 dots over 32, 1 dot over 35 Which box plot represents the dot plot data? box plot titled ACT Score with a minimum of 24, quartile 1 of 25, median of 26, quartile 3 of 29, and maximum of 35 box plot titled ACT Score with a minimum of 23, quartile 1 of 25, median of 26, quartile 3 of 29, and maximum of 36 box plot titled ACT Score with a minimum of 23, quartile 1 of 27, median of 30, quartile 3 of 34, and maximum of 36 box plot titled ACT Score with a minimum of 24, quartile 1 of 27, median of 28, quartile 3 of 30, and maximum of 35

Answers

Answer:

box plot titled ACT Score with a minimum of 24, quartile 1 of 27, median of 28, quartile 3 of 30, and maximum of 35

Step-by-step explanation:

The scores of the students represented on the dot plot are:

1 dot => 24

3 dots => 26, 26, 26

3 dots => 27, 27, 27

5 dots => 28, 28, 28, 28, 28

3 dots => 30, 30, 30

3 dots => 32, 32, 32

1 dot => 35

Quickly, we can ascertain 3 values from these data points of which we can use to find out which box plot represents the dot plot data.

The minimum score = 24

The maximum score = 35

The median score is the 10th value, which is the middle value of the data point = 28

Therefore, we can conclude that: "box plot titled ACT Score with a minimum of 24, quartile 1 of 27, median of 28, quartile 3 of 30, and maximum of 35".

Look at the chore chart--write a notice and a wonder about the chart. Click on the image to see the chart. Enter ur answer

Answers

Answer:

I noticed that to babysit my cousin was the chore that doled out the most, and I wonder why pet my dog is even a chore. Do they not love their pets?

What is the simplified form of the following expression? 2 StartRoot 18 EndRoot + 3 StartRoot 2 EndRoot + StartRoot 162 EndRoot 6 StartRoot 2 EndRoot 18 StartRoot 2 EndRoot 30 StartRoot 2 EndRoot 36 StartRoot 2 EndRoot

Answers

Answer:

[tex]18\sqrt2[/tex]

Step-by-step explanation:

To simplify:

[tex]2 \sqrt{18}+ 3 \sqrt2+ \sqrt{162 }[/tex]

First of all, let us write 18 and 162 as product of prime factors:

[tex]18 = 2 \times \underline{3 \times 3}\\162 = 2 \times \underline{3 \times 3} \times \underline{3 \times 3}[/tex]

The pairs are underlined as above.

While taking roots, only one of the numbers from the pairs will be chosen.

Now, taking square roots.

[tex]\sqrt{18} =3 \sqrt2[/tex]

[tex]162 = 3 \times 3 \times \sqrt 2 = 9 \sqrt2[/tex]

So, the given expression becomes:

[tex]2 \sqrt{18}+ 3 \sqrt2+ \sqrt{162 } = 2 \times 3\sqrt2 + 3\sqrt2 +9\sqrt2\\\Rightarrow 6\sqrt2 + 3\sqrt2 +9\sqrt2\\\Rightarrow \sqrt2(6+3+9)\\\Rightarrow \bold{18\sqrt2}[/tex]

So, the answer is:

[tex]18\sqrt2[/tex] or 18 StartRoot 2 EndRoot

Answer:

its B. 18 sqrt(2)

Step-by-step explanation:

just took test

A population consists of 100 elements. We want to draw a simple, random sample of 20 elements from this population. On the first selection, the probability of any particular element being selected is ____.

Answers

Answer:

1/5

Step-by-step explanation:

Probability is the likelihood or chance that an event will occur.

Probability = expected outcome of event /total outcome

Since the population consists of 100 elements, the total outcome of event = 100.

If random sample of 20 element is drawn from the population, the expected outcome = 20

On the first selection, the probability of any particular element being selected = 20/100 = 1/5

The average person lives for about 78 years. Does the average person live for at least 1,000,000 days? (Hint: There are 367 days in each year.)
what i

Answers

Answer:

[tex]\large \boxed{\sf No}[/tex]

Step-by-step explanation:

There are 365 days in 1 year.

The average person lives for about 78 years.

Multiply 78 by 365 to find the value in days.

[tex]78 \times 365= 28470[/tex]

The average person lives for about 28470 days.

Answer: No

Explanation:

78 x 367 = 28,626 days

And 28,626 days < 1,000,000 days

So an average person cannot live 1,000,000 days

Hope this helps!

please help solving.​

Answers

Answer:

right machine first, then left.6 into left machine, then right

Step-by-step explanation:

a) Putting 6 into the first (left) machine will give an output of ...

  y = √(6 -5) = √1 = 1

Putting 1 into the second (right) machine will give an output of ...

  y = 1² -6 = -5

This answers the second question, but not the first question.

__

If we put 6 into the right machine first, we get an output of ...

  y = 6² -6 = 30

Putting 30 into the left machine, we get an output of ...

  y = √(30 -5) = √25 = 5 . . . . . the desired output.

The input must go into the right machine first, then its output goes into the left machine to get a final output of 5 from an input of 6.

__

b) The left machine cannot produce negative outputs, so achieving an output of -5 with the arrangement used in part A is not possible. (green curves in the attached graph)

However, as we have shown above, inputting 6 to the left machine first, following that by processing with the right machine, can produce an output of -5. (purple curve in the attached graph)

If f(x)=x/2-3and g(x)=4x^2+x-4, find (f+g)(x)

Answers

Step-by-step explanation:

(f+g)(x) = f(x) + g(x)

= x/2-3 + 4x²+x+4

= ..........

Other Questions
HELP ASAP! 50 POINTS! Which type of electromagnetic radiation has a longer wavelength than visible light? (4 points) Gamma rays Infrared radiation Ultraviolet radiation Xrays data related to the inventories of alpine ski equipment and supplis is presented below 180000 the inventory of skis would be valued at She said, Could you write that down for me?. (change into indirect speech) Wish!! I would get this answer as fastly as possible... Remember, Sure to mark you as the brainliest... A force of pounds makes an angle of with a second force. The resultant of the two forces makes an angle of to the first force. Find the magnitudes of the second force and of the resultant. Question 1(Multiple Choice Worth 5 points) (07.02 LC) Celine is playing a game at the school carnival. There is a box of marbles, and each box has a white, a green, a blue, and an orange marble. There is also a fair 12-sided die labeled with the numbers 1 through 12. How many outcomes are in the sample space for pulling a marble out of the box and rolling the die? 48 32 16 8 Deshaun bought 6 bags of sugar for his restaurant. Each bag weighed 6.9 kilograms. How many kilograms of sugar did he buy total? During the first month of operations ended August 31, Kodiak Fridgeration Company manufactured 60,000 mini refrigerators, of which 54,000 were sold. Operating data for the month are summarized as follows:Sales $10,260,000.00 Manufacturing costs: Direct materials $5,100,000.00 Direct labor 1,800,000.00 Variable manufacturing cost 1,200,000.00 Fixed manufacturing cost 840,000.00 8,940,000.00 Selling and administrative expenses: Variable $972,000.00 324,000.00 Fixed 1,296,000.00 Required: 1. Prepare an income statement based on the absorption costing concept.2. Prepare an income statement based on the variable costing concept.3. Explain the reason for the difference in the amount of operating income reported in (1) and (2). Refer to the list of Labels and Amount Descriptions providedLabels and Amount Descriptions Labels August 31 Cost of goods sold Fixed costs For the Month Ended August 31 Variable cost of goods sold Amount Descriptions Contribution margin Contribution margin ratio Cost of goods manufactured Fixed manufacturing costs Fixed selling and administrative expenses Gross profit Operating income Inventory, August 31 Loss from operations Manufacturing margin Planned contribution margin Sales Sales mix Selling and administrative expenses Total cost of goods sold Total fixed costs Total fixed costs Total variable cost of goods sold Variable cost of goods manufactured Variable selling and administrative expenses Which of the following is NOT true? A. 5x + 6x = 70 degreesB. 5x + 6x < 180 degreesC. 5x + 6x = 110 degreesD. 5x + 6x + 70 degrees = 180 degreesPlease include ALL work! Four of the five commissioners for the Federal Trade Commission were working late one evening in their Washington, D.C. offices. The four were catching up on their electronic mail. The four soon discovered each others' presence on electronic mail. One commissioner wrote to the other three, "Let's get some business done so long as we're all here. We could resolve that unfair competition case that's pending for next month. We can iron out our differences via e-mail." The electronic exchanges of the commissioners would: 2. Why do you think mandatory minimum punishment sentencing dont have the deterrenteffect that they were created to prove? IM SO CONFUSED SOMEONE HELPP!! is it A? i honestly dont even know anymore bc everytime it says im wrong :((((( The Fundamental Orders of Connecticut was The left end of a long glass rod 8.00 cm in diameter and with an index of refraction of 1.60 is ground and polished to a convex hemispherical surface with a radius of 4.00 cm. An object in the form of an arrow 1.70 mm tall, at right angles to the axis of the rod, is located on the axis 24.0 cm to the left of the vertex of the convex surface.A) Find the position of the image of the arrow formed by paraxial rays incident on the convex surface.B) Find the height of the image formed by paraxial rays incident on the convex surface. C) Is the image erect or inverted? A square has a perimeter of 32 and one vertex at (6,8). what could be the location of another vertex of this shape? PLEASE HELP I AM IN A RUSH!!! On a coordinate plane, line M N goes through (negative 4, 0) and (4, 2). Point P is at (2, negative 4). Which point on the x-axis lies on the line that passes through point P and is perpendicular to line MN? (0, 1) (0, 4) (1, 0) (4, 0) You make a pattern bydrawing three similar rectangles. The widthof the smallest rectangle is45of the width ofthe medium-sized rectangle. The width ofthe medium-sized rectangle is45of thewidth of the largest rectangle. The largestrectangle is 12 inches long and 8 incheswide. Find the dimensions of the smallestrectangle. Explain your reasoning. Durante la explosin de una planta nuclear como la ocurrida en Chernbyl en 1986, una persona se expone por un tiempo a esta radiacin qu daos a nivel molecular (ADN O ARN) podra sufrir? Y cmo afectara esto su salud y a la de su descendencia? What is the goal of the scientific method how does liberty's daughters by mary beth norton end Two spheres A and B of negligible dimensions and masses 1 kg and 3 kg respectively, are supported on the smooth circular surface, fixed to the ground with a centre O and radius of 0.1m. The spheres are joined by the cord shown in length /20 m; determine the angles and corresponding to the position of equilibrium of the spheres with respect to the vertical passing through O.